Anda di halaman 1dari 26

iii. Abatement of Nuisance Estate of Gregoria Grancisco v CA Facts: A quonset was constructed by the American Liberation Forces in 1944.

It was purchased in 1946 by Gregoria Francisco. It stands on a lot owned by the PPA and faces the municipal wharf. By virtue of Proclamation No. 83 issued by President Elpidio Quirino, said land was declared for the exclusive use of port facilities. The PPA issued to Tan Gin San, spouse of Gregoria Francisco, a permit to occupy the lot where the building stands for a period of one (1) year, to expire on 31 December 1989. The permittee was using the quonset for the storage of copra. On May 1989, the Mayor notified Tan Gin San to remove or relocate its Quonset building citing Ordinance No. 147, noting its antiquated and dilapidated structure; and stressing the "clean-up campaign on illegal squatters and unsanitary surroundings along Strong Boulevard." Since the notifications remained unheeded, the Mayor ordered the demolition on 24 May 1989. Petitioner sought a Writ of Prohibition with Injunction and Damages before the RTC of Basilan. The RTC denied the writ and upheld the power of the Mayor to order the demolition without judicial authority pursuant to Ordinance 147. On 6 September 1989, petitioner's quonset building was completely demolished. In its place sprang shanties and nipa huts. The CA reversed the RTC and ruled that the mayor was not vested with power to order summarily without any judicial proceeding to demolish the Quonset building which was not a nuisance per se. However, upon reconsideration, the CA reversed itself and ruled that the deficiency was remedied when petitioner filed a petition for prohibition and injunction and was heard on oral argument. Issue: WON the Mayor could summarily, without judicial process, order the demolition of petitioner's quonset building. Held: No Ratio: Ordinance No. 147, enacted on 27 December 1977, and relied upon by respondents, is entitled "An Ordinance Establishing Comprehensive Zoning Regulations for the Municipality of Isabela." It is not disputed that the quonset building, which is being used for the storage of copra, is located outside the zone for warehouses. It is referred to in Ordinance as a non-conforming structure, which should be relocated. And in the event that an immediate relocation of the building can not be accomplished, Sec 16 of the Ordinance provides: A certificate of non-conformance for all non-conforming uses shall be applied for by the owner or agent of the property involved within 12mo from the approval of this Ordinance, otherwise the nonconforming use may be condemned or removed at the owner's expense. Even granting that petitioner failed to apply for a Certificate of Non-conformance, the provision should not be interpreted as authorizing the summary removal of a non-conforming building by the municipal government. For if it does, it must be struck down for being in contravention of the requirements of due process, as originally held by the CA. Moreover, the enforcement and administration of the provisions of the Ordinance resides with the Zoning Administrator . It is said official who may call upon the City Fiscal to institute the necessary legal proceedings to enforce the provisions of the Ordinance. And any person aggrieved by the decision of the Zoning Administrator regarding the enforcement of the Ordinance may appeal to the Board of Zoning Appeals. That a summary remedy can not be resorted to is further evident from the penal provisions. Violation of a municipal ordinance neither empowers the Municipal Mayor to avail of extra-judicial remedies. On the contrary, the Local Government Code imposes upon him the duty "to cause to be instituted judicial proceedings in connection with the violation of ordinances". Respondents can not seek cover under the general welfare clause authorizing the abatement of nuisances without judicial proceedings. That tenet applies to a nuisance per se or one which affects the immediate safety of persons and property and may be summarily abated under the undefined law of necessity. The

storage of copra in the quonset building is a legitimate business. By its nature, it can not be said to be injurious to rights of property, of health or of comfort of the community. If it be a nuisance per accidens it may be so proven in a hearing conducted for that purpose. It is not per se a nuisance warranting its summary abatement without judicial intervention. While the Sangguniang Bayan may provide for the abatement of a nuisance (Local Government Code, Sec. 149 (ee) ), it can not declare a particular thing as a nuisance per se and order its condemnation. The nuisance can only be so adjudged by judicial determination. Petitioner was in lawful possession of the lot and quonset building by virtue of a permit from the PPA when demolition was effected. It was not squatting on public land. Its property was not of trifling value. It was entitled to an impartial hearing before a tribunal authorized to decide whether the quonset building did constitute a nuisance in law. There was no compelling necessity for precipitate action. It follows then that the public officials of Isabela, Basilan, transcended their authority in abating summarily petitioner's quonset building. They had deprived petitioner of its property without due process of law. The fact that petitioner filed a suit for prohibition and was subsequently heard thereon will not cure the defect, as opined by the CA, the demolition having been a fait accompli prior to hearing and the authority to demolish without a judicial order being a prejudicial issue Technology Developers, Inc v CA Facts: Petitioner received a letter from private respondent acting mayor Pablo N. Cruz, ordering the full cessation of the operation of the petitioner's plant located at Guyong, Sta. Maria, Bulacan. The letter requested Plant Manager Armando Manese to bring with him to the office of the mayor on February 20, 1989 the following: a) Building permit; b) Mayor's permit; c) Region III-Pollution of Environment and Natural Resources Anti-Pollution Permit; and of other document. At the requested conference, petitioner undertook to comply with respondent's request for the production of the required documents. Petitioner commenced to secure "Region III-DENR Anti-Pollution Permit," although among the permits previously secured prior to the operation of petitioner's plant was a "Temporary Permit to Operate Air Pollution Installation" issued by the then National Pollution Control Commission and is now at a stage where the Environmental Management Bureau is trying to determine the correct kind of antipollution devise to be installed as part of petitioner's request for the renewal of its permit. Petitioner's attention having been called to its lack of mayor's permit, it sent its representatives to the office of the mayor to secure the same but were not entertained. On April 6, 1989, without previous and reasonable notice upon petitioner, respondent ordered the Municipality's station commander to padlock the premises of petitioner's plant, thus effectively causing the stoppage of its operation. Petitioner instituted an action for certiorari, prohibition, mandamus with preliminary injunction against private respondent. The judge found that petitioner is entitled to the issuance of a writ of preliminary injunction upon posting of a bond worth P50,000. During the MR, the Provincial Prosecutor presented his evidence prepared by Marivic Guina, Due to the manufacturing process and nature of raw materials used, the fumes coming from the factory may contain particulate matters which are hazardous to the health of the people. As such, the company should cease operating until such a time that the proper air pollution device is installed and operational." The lower court then set aside the order which granted a writ of preliminary mandatory injunction and dissolved the writ issued. Issue: WON the writ of preliminary injunction should be granted Held: No Ratio: The matter of issuance of a writ of preliminary injunction is addressed to the sound judicial discretion of the trial court and its action shall not be disturbed on appeal unless it is demonstrated that it acted without jurisdiction or in excess of jurisdiction or otherwise, in grave abuse of its discretion. By the same token the court that issued such a preliminary relief may recall or dissolve the writ as the circumstances may warrant.

The following circumstances militate against the maintenance of the writ of preliminary injunction sought by petitioner: 1. No mayor's permit had been secured. While it is true that the matter of determining whether there is a pollution of the environment that requires control if not prohibition of the operation of a business is addressed to the National Pollution Control Commission of the Ministry of Human Settlements, now the Environmental Management Bureau, it must be recognized that the mayor of a town has as much responsibility to protect its inhabitants from pollution, and by virtue of his police power, he may deny the application for a permit to operate a business or otherwise close the same unless appropriate measures are taken to control and/or avoid injury to the health of the residents of the community from the emissions in the operation of the business. 2. The Acting Mayor called the attention of petitioner to the pollution emitted by the fumes of its plant whose offensive odor "not only pollute the air in the locality but also affect the health of the residents in the area," so that petitioner was ordered to stop its operation until further orders and it was required to bring the following: (1) Building permit; (2) Mayor's permit; and (3) Region III-DENR Anti-Pollution permit. 3. This action of the Acting Mayor was in response to the complaint of the residents of Barangay Guyong, Sta. Maria, Bulacan, directed to the Provincial Governor through channels. The NBI finding that some of the signatures in the 4-page petition were written by one person, appears to be true in some instances, (particularly as among members of the same family), but on the whole the many signatures appear to be written by different persons. The certification of the barrio captain of said barrio that he has not received any complaint on the matter must be because the complaint was sent directly to the Governor through the Acting Mayor. 4. The closure order of the Acting Mayor was issued only after an investigation was made by Marivic Guina who in her report observed that the fumes emitted by the plant goes directly to the surrounding houses and that no proper air pollution device has been installed. 5. Petitioner failed to produce a building permit from the municipality of Sta. Maria, but instead presented a building permit issued by an official of Makati on March 6, 1987. 6. While petitioner was able to present a temporary permit to operate by the then National Pollution Control Commission on December 15, 1987, the permit was good only up to May 25, 1988. Petitioner had not exerted any effort to extend or validate its permit much less to install any device to control the pollution and prevent any hazard to the health of the residents of the community. All these factors justify the dissolution of the writ of preliminary injunction by the trial court and the appellate court correctly upheld the action of the lower court. Petitioner takes note of the plea of petitioner focusing on its huge investment in this dollar-earning industry. It must be stressed however, that concomitant with the need to promote investment and contribute to the growth of the economy is the equally essential imperative of protecting the health, nay the very lives of the people, from the deleterious effect of the pollution of the environment. Laguna Lake Development Authority v CA Facts: RA 4850 was enacted creating the "Laguna Lake Development Authority." This agency was supposed to accelerate the development and balanced growth of the Laguna Lake area and the surrounding provinces, cities and towns, in the act, within the context of the national and regional plans and policies for social and economic development. PD 813 amended certain sections RA 4850 because of the concern for the rapid expansion of Metropolitan Manila, the suburbs and the lakeshore towns of Laguna de Bay, combined with current and prospective uses of the lake for municipal-industrial water supply, irrigation, fisheries, and the like. To effectively perform the role of the Authority under RA 4850, the Chief Executive issued EO 927 further defined and enlarged the functions and powers of the Authority and named and enumerated the towns, cities

and provinces encompassed by the term "Laguna de Bay Region". Also, pertinent to the issues in this case are the following provisions of EO 927 which include in particular the sharing of fees: Sec 2: xxx the Authority shall have exclusive jurisdiction to issue permit for the use of all surface water for any projects or activities in or affecting the said region including navigation, construction, and operation of fishpens, fish enclosures, fish corrals and the like. SEC. 3. Collection of Fees. The Authority is hereby empowered to collect fees for the use of the lake water and its tributaries for all beneficial purposes including but not limited to fisheries, recreation, municipal, industrial, agricultural, navigation, irrigation, and waste disposal purpose; Provided, that the rates of the fees to be collected, and the sharing with other government agencies and political subdivisions, if necessary, shall be subject to the approval of the President of the Philippines upon recommendation of the Authority's Board, except fishpen fee, which will be shared in the following manner: 20 percent of the fee shall go to the lakeshore local governments, 5 percent shall go to the Project Development Fund which shall be administered by a Council and the remaining 75 percent shall constitute the share of LLDA. However, after the implementation within the three-year period of the Laguna Lake Fishery Zoning and Management Plan the sharing will be modified as follows: 35 percent of the fishpen fee goes to the lakeshore local governments, 5 percent goes to the Project Development Fund and the remaining 60 percent shall be retained by LLDA; Provided, however, that the share of LLDA shall form part of its corporate funds and shall not be remitted to the National Treasury as an exception to the provisions of Presidential Decree No. 1234. Then came Republic Act No. 7160. The municipalities in the Laguna Lake Region interpreted the provisions of this law to mean that the newly passed law gave municipal governments the exclusive jurisdiction to issue fishing privileges within their municipal waters because R.A. 7160 provides: "Sec. 149. Fishery Rentals; Fees and Charges (a) Municipalities shall have the exclusive authority to grant fishery privileges in the municipal waters and impose rental fees or charges therefor in accordance with the provisions of this Section. Municipal governments thereupon assumed the authority to issue fishing privileges and fishpen permits. Big fishpen operators took advantage of the occasion to establish fishpens and fishcages to the consternation of the Authority. Unregulated fishpens and fishcages occupied almost one-third the entire lake water surface area, increasing the occupation drastically from 7,000 ha in 1990 to almost 21,000 ha in 1995. The Mayor's permit to construct fishpens and fishcages were all undertaken in violation of the policies adopted by the Authority on fishpen zoning and the Laguna Lake carrying capacity. In view of the foregoing circumstances, the Authority served notice to the general public that: 1. All fishpens, fishcages and other aqua -culture structures in the Laguna de Bay Region, which were not registered or to which no application for registration and/or permit has been filed with Laguna Lake Development Authority as of March 31, 1993 are hereby declared outrightly as illegal. 2. All fishpens; fishcages and other aqua-culture structures so declared as illegal shall be subject to demolition which shall be undertaken by the Presidential Task Force for illegal Fishpen and Illegal Fishing. 3. Owners of fishpens, fishcages and other aqua-culture structures declared as illegal shall, without prejudice to demolition of their structures be criminally charged in accordance with Section 39-A of Republic Act 4850 as amended by P.D. 813 for violation of the same laws. Violations of these laws carries a penalty of imprisonment of not exceeding 3 years or a fine not exceeding Five Thousand Pesos or both at the discretion of the court. All operators of fishpens, fishcages and other aqua-culture structures declared as illegal in accordance with the foregoing Notice shall have one (1) month on or before 27 October 1993 to show cause before the LLDA why their said fishpens, fishcages and other aqua-culture structures should not be demolished/dismantled." One month, thereafter, the Authority sent notices to the concerned owners of the illegally constructed fishpens, fishcages and other aqua-culture structures advising them to dismantle their respective structures within 10 days from receipt thereof, otherwise, demolition shall be effected.

The fishpen owners filed injunction cases against the LLDA. The LLDA filed motions to dismiss the cases against it on jurisdictional grounds. The motions to dismiss were denied. Meanwhile, TRO/writs of preliminary mandatory injunction were issued enjoining the LLDA from demolishing the fishpens and similar structures in question. Hence, the present petition for certiorari, prohibition and injunction. The CA dismissed the LLDAs consolidated petitions. It ruled that (A) LL DA is not among those quasi-judicial agencies of government appealable only to the Court of Appeals; (B) the LLDA charter does vest LLDA with quasi-judicial functions insofar as fishpens are concerned; (C) the provisions of the LLDA charter insofar as fishing privileges in Laguna de Bay are concerned had been repealed by the Local Government Code of 1991; (D) in view of the aforesaid repeal, the power to grant permits devolved to respective local government units concerned. Issue: Which agency of the Government - the LLDA or the towns and municipalities comprising the region should exercise jurisdiction over the Laguna Lake and its environs insofar as the issuance of permits for fishery privileges is concerned? Held: LLDA Ratio: Section 4 (k) of RA 4850, the provisions of PD 813, and Section 2 of EO 927, specifically provide that the LLDA shall have exclusive jurisdiction to issue permits for the use or all surface water for any projects or activities in or affecting the said region, including navigation, construction, and operation of fishpens, fish enclosures, fish corrals and the like. On the other hand, RA 7160 has granted to the municipalities the exclusive authority to grant fishery privileges in municipal waters. The Sangguniang Bayan may grant fishery privileges to erect fish corrals, oyster, mussels or other aquatic beds or bangus fry area within a definite zone of the municipal waters. The provisions of RA7160 do not necessarily repeal the laws creating the LLDA and granting the latter water rights authority over Laguna de Bay and the lake region. The Local Government Code of 1991 does not contain any express provision which categorically expressly repeal the charter of the Authority. It has to be conceded that there was no intent on the part of the legislature to repeal Republic Act No. 4850 and its amendments. The repeal of laws should be made clear and expressed. It has to be conceded that the charter of the LLDA constitutes a special law. RA 7160 is a general law. It is basic is basic in statutory construction that the enactment of a later legislation which is a general law cannot be construed to have repealed a special law. It is a well-settled rule in this jurisdiction that "a special statute, provided for a particular case or class of cases, is not repealed by a subsequent statute, general in its terms, provisions and application, unless the intent to repeal or alter is manifest, although the terms of the general law are broad enough to include the cases embraced in the special law." Where there is a conflict between a general law and a special statute, the special statute should prevail since it evinces the legislative intent more clearly that the general statute. The special law is to be taken as an exception to the general law in the absence of special circumstances forcing a contrary conclusion. This is because implied repeals are not favored and as much as possible, given to all enactments of the legislature. A special law cannot be repealed, amended or altered by a subsequent general law by mere implication. Considering the reasons behind the establishment of the Authority, which are enviromental protection, navigational safety, and sustainable development, there is every indication that the legislative intent is for the Authority to proceed with its mission. We are on all fours with the manifestation of LLDA that "Laguna de Bay, like any other single body of water has its own unique natural ecosystem. The 900 km lake surface water, the 8 major river tributaries and several other smaller rivers that drain into the lake, the 2,920 km2 basin or watershed transcending the boundaries of Laguna and Rizal provinces, constitute one integrated delicate natural ecosystem that needs to be protected with uniform set of policies; if we are to be serious in our aims of attaining sustainable development. This is an exhaustible natural resource-a very limited one-which requires judicious management and optimal utilization to ensure renewability and preserve its ecological integrity and balance.

Managing the lake resources would mean the implementation of a national policy geared towards the protection, conservation, balanced growth and sustainable development of the region with due regard to the inter-generational use of its resources by the inhabitants in this part of the earth. The authors of Republic Act 4850 have foreseen this need when they passed this LLDA law-the special law designed to govern the management of our Laguna de Bay lake resources. Laguna de Bay therefore cannot be subjected to fragmented concepts of management policies where lakeshore local government units exercise exclusive dominion over specific portions of the lake water. The implementation of a cohesive and integrated lake water resource management policy, therefore, is necessary to conserve, protect and sustainably develop Laguna de Bay." The power of the LGUs to issue fishing privileges was clearly granted for revenue purposes. This is evident from the fact that Section 149 of the New Local Government Code empowering local governments to issue fishing permits is embodied in Chapter 2, Book II, of Republic Act No. 7160 under the heading, "Specific Provisions On The Taxing And Other Revenue Raising Power of LGUs. On the other hand, the power of the Authority to grant permits for fishpens, fishcages and other aqua-culture structures is for the purpose of effectively regulating and monitoring activities in the Laguna de Bay region and for lake quality control and management. 6 It does partake of the nature of police power which is the most pervasive, the least limitable and the most demanding of all State powers including the power of taxation. Accordingly the charter of the Authority which embodies a valid exercise of police power should prevail over the Local Government Code of 1991 on matters affecting Laguna de Bay. There should be no quarrel over permit fees for fishpens, fishcages and other aqua-culture structures in the Laguna de Bay area. Section 3 of Executive Order No. 927 provides for the proper sharing of fees collected. In respect to the question as to whether the Authority is a quasi-judicial agency or not, it is our holding that, considering the provisions of Section 4 of Republic Act No. 4850 and Section 4 of Executive Order No. 927, series of 1983, and the ruling of this Court in Laguna Lake Development Authority vs. Court of Appeals, there is no question that the Authority has express powers as a regulatory a quasi-judicial body in respect to pollution cases with authority to issue a "cease a desist order" and on matters affecting the construction of illegal fishpens, fishcages and other aqua-culture structures in Laguna de Bay. The Authority's pretense, however, that it is co-equal to the Regional Trial Courts such that all actions against it may only be instituted before the Court of Appeals cannot be sustained. On actions necessitating the resolution of legal questions affecting the powers of the Authority as provided for in its charter, the Regional Trial Courts have jurisdiction. In view of the foregoing, this Court holds that Section 149 of RA 7160, otherwise known as the Local Government Code of 1991, has not repealed the provisions of the charter of the LLDA, Republic Act No. 4850, as amended. Thus, the Authority has the exclusive jurisdiction to issue permits for the enjoyment of fishery privileges in Laguna de Bay to the exclusion of municipalities situated therein and the authority to exercise such powers as are by its charter vested on it. iv. Power of Eminent Domain Section 19. Eminent Domain. - A local government unit may, through its chief executive and acting pursuant to an ordinance, exercise the power of eminent domain for public use, or purpose or welfare for the benefit of the poor and the landless, upon payment of just compensation, pursuant to the provisions of the Constitution and pertinent laws: Provided, however, That the power of eminent domain may not be exercised unless a valid and definite offer has been previously made to the owner, and such offer was not accepted: Provided, further, That the local government unit may immediately take possession of the property upon the filing of the expropriation proceedings and upon making a deposit with the proper court of at least fifteen percent (15%) of the fair market value of the property based on the current tax declaration of the property to be expropriated: Provided, finally, That, the amount to be paid for the expropriated property shall be determined by the proper court, based on the fair market value at the time of the taking of the property. Moday et al v. Court of Appeals

Facts: The Sangguniang Bayan of the Municipality of Bunawan in Agusan del Sur passed Resolution No. 4389, "Authorizing the Municipal Mayor to Initiate the Petition for Expropriation of a One (1) Hectare Portion of Lot No. 6138-Pls-4 Along the National Highway Owned by Percival Moday for the Site of Bunawan Farmers Center and Other Government Sports Facilities." The Resolution was approved by Mayor Anuncio Bustillo and was transmitted to the Sangguniang Panlalawigan for its approval. The Sangguniang Panlalawigan disapproved said Resolution and returned it with the comment that "expropriation is unnecessary considering that there are still available lots in Bunawan for the establishment of the government center." The municipality filed a petition for eminent domain against Percival Moday before the RTC. The municipality then filed a motion to take or enter upon the possession of the land upon deposit with the municipal treasurer of the required amount. The RTC granted the motion. It ruled that the Sangguniang Panlalawigan's failure to declare the resolution invalid leaves it effective. It added that the duty of the Sangguniang Panlalawigan is merely to review the ordinances and resolutions passed by the Sangguniang Bayan under Section 208 (1) of B.P. Blg. 337, old Local Government Code and that the exercise of eminent domain is not one of the acts enumerated in Section 19 requiring the approval of the Sangguniang Panlalawigan. Petitioners elevated the case in a petition for certiorari before the CA. The CA held that the public purpose for the expropriation is clear from Resolution No. 43-89 and that since the Sangguniang Panlalawigan of Agusan del Sur did not declare Resolution No. 43-89 invalid, expropriation of petitioners' property could proceed. Meanwhile, the Municipality had erected three buildings on the subject property: the Association of Barangay Councils (ABC) Hall, the Municipal Motorpool, both wooden structures, and the Bunawan Municipal Gymnasium, which is made of concrete. In the instant petition for review, petitioner seeks the reversal of the decision and resolution of the CA and a declaration that Resolution No. 43-89 of the Municipality of Bunawan is null and void. Issue: WON a municipality may expropriate private property by virtue of a municipal resolution which was disapproved by the Sangguniang Panlalawigan. Held: Yes Ratio: Eminent domain, the power which the Municipality of Bunawan exercised in the instant case, is a fundamental State power that is inseparable from sovereignty. It is government's right to appropriate, in the nature of a compulsory sale to the State, private property for public use or purpose. Inherently possessed by the national legislature, the power of eminent domain may be validly delegated to local governments, other public entities and public utilities. For the taking of private property by the government to be valid, the taking must be for public use and there must be just compensation. The Municipality's power to exercise the right of eminent domain is not disputed as it is expressly provided for BP 337, the local Government Code in force at the time expropriation proceedings were initiated. What petitioners question is the lack of authority of the municipality to exercise this right since the Sangguniang Panlalawigan disapproved Resolution No. 43-89. The Sangguniang Panlalawigan's disapproval of Resolution No. 43-89 is an infirm action which does not render said resolution null and void. The law, Section 153 of B.P. Blg. 337, grants the Sangguniang Panlalawigan the power to declare a municipal resolution invalid on the sole ground that it is beyond the power of the Sangguniang Bayan or the Mayor to issue. Velazco v. Blas: The only ground upon which a provincial board may declare any municipal resolution, ordinance, or order invalid is when such resolution, ordinance, or order is "beyond the powers conferred upon the council or president making the same." Absolutely no other ground is recognized by the law. A strictly legal question is before the provincial board in its consideration of a municipal resolution, ordinance, or order. The provincial disapproval of any resolution, ordinance, or order must be premised specifically upon the fact that such resolution, ordinance, or order is outside the scope of the legal powers conferred by law. If a provincial board passes these limits, it usurps the legislative function of the municipal council or president. Such has been the consistent course of executive authority. Thus, the Sangguniang Panlalawigan

was without the authority to disapprove Municipal Resolution No. 43-89 for the Municipality of Bunawan clearly has the power to exercise the right of eminent domain and its Sangguniang Bayan the capacity to promulgate said resolution, pursuant to the earlier-quoted Section 9 of B.P. Blg. 337. Perforce, it follows that Resolution No. 43-89 is valid and binding and could be used as lawful authority to petition for the condemnation of petitioners' property. As regards the accusation of political oppression, it is alleged that Moday incurred the ire of then Mayor Bustillo when he refused to support the latter's candidacy for mayor in previous elections. Petitioners claim that then incumbent Mayor Bustillo used the expropriation to retaliate by expropriating their land even if there were other properties belonging to the municipality and available for the purpose. Specifically, they allege that the municipality owns a vacant seven-hectare property adjacent to petitioners' land, evidenced by a sketch plan. The limitations on the power of eminent domain are that the use must be public, compensation must be made and due process of law must be observed. The Supreme Court, taking cognizance of such issues as the adequacy of compensation, necessity of the taking and the public use character or the purpose of the taking, has ruled that the necessity of exercising eminent domain must be genuine and of a public character. Government may not capriciously choose what private property should be taken. Prov. Of Camarines Sur v. Court of Appeals Facts: The Sangguniang Panlalawigan of Camarines Sur passed Resolution No. 129, Series of 1988, authorizing the Provincial Governor to purchase or expropriate property contiguous to the provincial capitol site, in order to establish a pilot farm for non-food and non-traditional agricultural crops and a housing project for provincial government employees. Pursuant to the Resolution, the Province through Governor Luis R.Villafuerte, filed two cases for expropriation against Ernesto N. San Joaquin and Efren N. San Joaquin. The San Joaquins moved to dismiss the complaints on the ground of inadequacy of the price offered for their property. In an order, the trial court denied the motion to dismiss and authorized the Province to take possession of the property upon the deposit of P5,714.00. The trial court issued a writ of possession. The San Joaquins filed a motion for relief from the order and a motion to admit an amended motion to dismiss. Both motions were denied. In their petition before the Court of Appeals, the San Joaquins asked: (a) that Resolution No. 129, Series of 1988 be declared null and void; (b) that the complaints for expropriation be dismissed; and (c) that the order dated December 6, 1989 (i) denying the motion to dismiss and (ii) allowing the Province to take possession of the property subject of the expropriation and the order dated February 26, 1990, denying the motion to admit the amended motion to dismiss, be set aside. They also asked that an order be issued to restrain the trial court from enforcing the writ of possession, and thereafter to issue a writ of injunction. The Province claimed that it has the authority to initiate the expropriation proceedings under Sections 4 and 7 of Local Government Code and that the expropriations are for a public purpose. The Solicitor General stated that under Section 9 of the Local Government Code, there was no need for the approval by the Office of the President of the exercise by the Sangguniang Panlalawigan of the right of eminent domain. However, the Solicitor General expressed the view that the Province of Camarines Sur must first secure the approval of the Department of Agrarian Reform of the plan to expropriate the lands of petitioners for use as a housing project. The CA set aside the order of the court and ordered the trial court to suspend the expropriation proceedings until the province shall have submitted the requisite approval of the DAR. Issue: WON the expropriation was proper Ratio: The CA did not rule on the validity of the questioned resolution; neither did it dismiss the complaints. However, when the CA ordered the suspension of the proceedings until the Province shall have obtained the authority of the DAR to change the classification of the lands sought to be expropriated from agricultural to

non-agricultural use, it assumed that the resolution is valid and that the expropriation is for a public purpose or public use. Public Purpose. Modernly, there has been a shift from the literal to a broader interpretation of "public purpose" or "public use" for which the power of eminent domain may be exercised. The old concept was that the condemned property must actually be used by the general public (e.g. roads, bridges, public plazas, etc.) before the taking thereof could satisfy the constitutional requirement of "public use". Under the new concept, "public use" means public advantage, convenience or benefit, which tends to contribute to the general welfare and the prosperity of the whole community, like a resort complex for tourists or housing project. The expropriation of the property authorized by the questioned resolution is for a public purpose. The establishment of a pilot development center would inure to the direct benefit and advantage of the people of the Province of Camarines Sur. Once operational, the center would make available to the community invaluable information and technology on agriculture, fishery and the cottage industry. Ultimately, the livelihood of the farmers, fishermen and craftsmen would be enhanced. The housing project also satisfies the public purpose requirement of the Constitution. As held in Sumulong v. Guerrero, "Housing is a basic human need. Shortage in housing is a matter of state concern since it directly and significantly affects public health, safety, the environment and in sum the general welfare." Eminent Domain vs CARL. It is the submission of the Province of Camarines Sur that its exercise of the power of eminent domain cannot be restricted by the provisions of the CARL, particularly Section 65, which requires the approval of the DAR before a parcel of land can be reclassified from an agricultural to a nonagricultural land. The CA, following the recommendation of the Solicitor General, held that the Province of Camarines Sur must comply with the provision of Section 65 of the CARK and must first secure the approval of the Department of Agrarian Reform of the plan to expropriate the lands of the San Joaquins. In Heirs of Juancho Ardana v. Reyes, while the Court said that there was "no need under the facts of this petition to rule on whether the public purpose is superior or inferior to another purpose or engage in a balancing of competing public interest," it upheld the expropriation after noting that petitioners had failed to overcome the showing that the taking of 8,970 sq m formed part of the resort complex. A fair and reasonable reading of the decision is that this Court viewed the power of expropriation as superior to the power to distribute lands under the land reform program. The Solicitor General denigrated the power to expropriate by the Province of Camarines Sur by stressing the fact that LGUs exercise such power only by delegation. It is true that local government units have no inherent power of eminent domain and can exercise it only when expressly authorized by the legislature. It is also true that in delegating the power to expropriate, the legislature may retain certain control or impose certain restraints on the exercise thereof by the local governments. While such delegated power may be a limited authority, it is complete within its limits. Moreover, the limitations on the exercise of the delegated power must be clearly expressed, either in the law conferring the power or in other legislations. Section 9 of B.P. Blg. 337 does not intimate in the least that local government, units must first secure the approval of the Department of Land Reform for the conversion of lands from agricultural to non-agricultural use, before they can institute the necessary expropriation proceedings. Likewise, there is no provision in the Comprehensive Agrarian Reform Law which expressly subjects the expropriation of agricultural lands by local government units to the control of the Department of Agrarian Reform. The closest provision of law that the CA could cite to justify the intervention of the DAR in expropriation matters is Section 65 of the CARL. The opening, adverbial phrase of the provision sends signals that it applies to lands previously placed under the agrarian reform program as it speaks of "the lapse of five (5) years from its award." The rules on conversion of agricultural lands found in Section 4 (k) and 5 (1) of Executive Order No. 129-A, Series of 1987, cannot be the source of the authority of the DAR to determine the suitability of a parcel of agricultural land for the purpose to which it would be devoted by the expropriating authority. While those rules vest on the Department of Agrarian Reform the exclusive authority to approve or disapprove conversions of agricultural lands for residential, commercial or industrial uses, such authority is limited to the applications for reclassification submitted by the land owners or tenant beneficiaries.

Statutes conferring the power of eminent domain to political subdivisions cannot be broadened or constricted by implication. To sustain the Court of Appeals would mean that the LGUs can no longer expropriate agricultural lands needed for the construction of roads, bridges, schools, hospitals, etc, without first applying for conversion of the use of the lands with the DAR, because all of these projects would naturally involve a change in the land use. In effect, it would then be the DAR to scrutinize whether the expropriation is for a public purpose or public use. Ordinarily, it is the legislative branch of the local government unit that shall determine whether the use of the property sought to be expropriated shall be public, the same being an expression of legislative policy. The courts defer to such legislative determination and will intervene only when a particular undertaking has no real or substantial relation to the public use. There is also an ancient rule that restrictive statutes, no matter how broad their terms are, do not embrace the sovereign unless the sovereign is specially mentioned as subject thereto. The Republic of the Philippines, as sovereign, or its political subdivisions, as holders of delegated sovereign powers, cannot be bound by provisions of law couched in general term. The fears of private respondents that they will be paid on the basis of the valuation declared in the tax declarations of their property, are unfounded. This Court has declared as unconstitutional the Presidential Decrees fixing the just compensation in expropriation cases to be the value given to the condemned property either by the owners or the assessor, whichever was lower. As held in Municipality of Talisay v. Ramirez, the rules for determining just compensation are those laid down in Rule 67 of the Rules of Court, which allow private respondents to submit evidence on what they consider shall be the just compensation for their property. Barangay San Roque vs. Heirs of Pastor Facts: Petitioner filed before the MTC of Talisay, Cebu a Complaint to expropriate a property of the respondents. In an Order, the MTC dismissed the Complaint on the ground of lack of jurisdiction. It reasoned that "[e) minent domain is an exercise of the power to take private property for public use after payment of just compensation. In an action for eminent domain, therefore, the principal cause of action is the exercise of such power or right. The fact that the action also involves real property is merely incidental. An action for eminent domain is therefore within the exclusive original jurisdiction of the Regional Trial Court and not with this Court." On appeal, the RTC dismissed the complaint, holding that an action for eminent domain affected title to real property; hence, the value of the property to be expropriated would determine whether the case should be filed before the MTC or the RTC. The RTC concluded that the action should have been filed before the MTC since the value of the subject property was less than P20,000. Aggrieved, petitioner appealed directly to this Court, raising a pure question of law. Respondents contend that the Complaint for Eminent Domain affects the title to or possession of real property. Thus, they argue that the case should have been brought before the MTC, pursuant to BP 129 as amended by Section 3 (3) of RA 7691. This law provides that MTCs shall have exclusive original jurisdiction over all civil actions that involve title to or possession of real property, the assessed value of which does not exceed twenty thousand pesos or, in civil actions in Metro Manila, fifty thousand pesos exclusive of interest, damages of whatever kind, attorneys fees, litigation expenses and costs. Issue: WON an expropriation suit is one incapable of pecuniary estimation and is therefore within the jurisdiction of the RTC Held: Yes Ratio: "A review of the jurisprudence of this Court indicates that in determining whether an action is one the subject matter of which is not capable of pecuniary estimation, this Court has adopted the criterion of first ascertaining the nature of the principal action or remedy sought. If it is primarily for the recovery of a sum of

money, the claim is considered capable of pecuniary estimation, and whether jurisdiction is in the municipal courts or in the courts of first instance would depend on the amount of the claim. However, where the basic issue is something other than the right to recover a sum of money, or where the money claim is purely incidental to, or a consequence of, the principal relief sought, like in suits to have the defendant perform his part of the contract (specific performance) and in actions for support, or for annulment of a judgment or to foreclose a mortgage, this Court has considered such actions as cases where the subject of the litigation may not be estimated in terms of money, and are cognizable exclusively by courts of first instance. The rationale of the rule is plainly that the second class cases, besides the determination of damages, demand an inquiry into other factors which the law has deemed to be more within the competence of courts of first instance, which were the lowest courts of record at the time that the first organic laws of the Judiciary were enacted allocating jurisdiction. In the present case, an expropriation suit does not involve the recovery of a sum of money. Rather, it deals with the exercise by the government of its authority and right to take private property for public use. In National Power Corporation v. Jocson, the Court ruled that expropriation proceedings have two phases: "The first is concerned with the determination of the authority of the plaintiff to exercise the power of eminent domain and the propriety of its exercise in the context of the facts involved in the suit. It ends with an order, if not of dismissal of the action, of condemnation declaring that the plaintiff has a lawful right to take the property sought to be condemned, for the public use or purpose described in the complaint, upon the payment of just compensation to be determined as of the date of the filing of the complaint. An order of dismissal, if this be ordained, would be a final one, of course, since it finally disposes of the action and leaves nothing more to be done by the Court on the merits. So, too, would an order of condemnation be a final one, for thereafter as the Rules expressly state, in the proceedings before the Trial Court, no objection to the exercise of the right of condemnation (or the propriety thereof) shall be filed or heard. "The second phase of the eminent domain action is concerned with the determination by the court of the just compensation for the property sought to be taken. This is done by the Court with the assistance of not more than three (3) commissioners. The order fixing the just compensation on the basis of the evidence before, and findings of, the commissioners would be final, too. It would finally dispose of the second stage of the suit, and leave nothing more to be done by the Court regarding the issue. " It should be stressed that the primary consideration in an expropriation suit is whether the government or any of its instrumentalities has complied with the requisites for the taking of private property. Hence, the courts determine the authority of the government entity, the necessity of the expropriation, and the observance of due process. In the main, the subject of an expropriation suit is the governments exercise of eminent domain, a matter that is incapable of pecuniary estimation. True, the value of the property to be expropriated is estimated in monetary terms, for the court is duty-bound to determine the just compensation for it. This, however, is merely incidental to the expropriation suit. Indeed, that amount is determined only after the court is satisfied with the propriety of the expropriation. Verily, the Court held in Republic of the Philippines v. Zurbano that "condemnation proceedings are within the jurisdiction of Courts of First Instance," the forerunners of the regional trial courts. The said case was decided during the effectivity of the Judiciary Act of 1948 which, like BP 129 in respect to RTCs, provided that courts of first instance had original jurisdiction over "all civil actions in which the subject of the litigation is not capable of pecuniary estimation." The 1997 amendments to the Rules of Court were not intended to change these jurisprudential precedents. We are not persuaded by respondents argument that the present action involves the title to or possession of a parcel of land. They cite the observation of retired Justice Jose Y. Feria, an eminent authority in remedial law, that condemnation or expropriation proceedings are examples of real actions that affect the title to or possession of a parcel of land.

Their reliance is misplaced. Justice Feria sought merely to distinguish between real and personal actions. His discussion on this point pertained to the nature of actions, not to the jurisdiction of courts. In fact, in his prebar lectures, he emphasizes that jurisdiction over eminent domain cases is still within the RTCs under the 1997 Rules. To emphasize, the question in the present suit is whether the government may expropriate private property under the given set of circumstances. The government does not dispute respondents title to or possession of the same. Indeed, it is not a question of who has a better title or right, for the government does not even claim that it has a title to the property. It merely asserts its inherent sovereign power to "appropriate and control individual property for the public benefit, as the public necessity, convenience or welfare may demand. Mun. of Paranaque vs. V.M. Realty Corp Facts: Pursuant to Sangguniang Bayan Resolution No. 93-95, Series of 1993, the Municipality of Paraaque filed a Complaint for expropriation against V.M. Realty Corporation, over two parcels of land. Allegedly, the complaint was filed "for the purpose of alleviating the living conditions of the underprivileged by providing homes for the homeless through a socialized housing project." Petitioner, pursuant to its Sangguniang Bayan Resolution No. 577, Series of 1991, previously made an offer to enter into a negotiated sale of the property with private respondent, which the latter did not accept. The RTC authorized petition to take possession of the subject property upon its deposit with the clerk of court of an amount equivalent to 15% of its fair market value. Private Respondent filed an answer alleging that (a) the complaint failed to state a cause of action because it was filed pursuant to a resolution and not to an ordinance as required by RA 7160; and (b) the cause of action, if any, was barred by a prior judgment or res judicata. On private respondent's motion, its Answer was treated as a motion to dismiss. The trial court dismissed the complaint. Issue: WON the resolution is different from the ordinance Held: Yes Ratio: Petitioner contends that a resolution approved by the municipal council for the purpose of initiating an expropriation case "substantially complies with the requirements of the law" because the terms "ordinance" and "resolution" are synonymous for "the purpose of bestowing authority [on] the local government unit through its chief executive to initiate the expropriation proceedings in court in the exercise of the power of eminent domain." Petitioner seeks to bolster this contention by citing Article 36, Rule VI of the IRR of the Local Government Code, which provides: "If the LGU fails to acquire a private property for public use, purpose, or welfare through purchase, the LGU may expropriate said property through a resolution of the Sanggunian authorizing its chief executive to initiate expropriation proceedings." The Court disagrees. The power of eminent domain is lodged in the legislative branch of government, which may delegate the exercise thereof to LGUs, other public entities and public utilities. An LGU may therefore exercise the power to expropriate private property only when authorized by Congress and subject to the latter's control and restraints imposed "through the law conferring the power or in other legislations." In this case, Section 19 of RA 7160, which delegates to LGUs the power of eminent domain, also lays down the parameters for its exercise. Thus, the following essential requisites must concur before an LGU can exercise the power of eminent domain: 1. An ordinance is enacted by the local legislative council authorizing the local chief executive, in behalf of the LGU, to exercise the power of eminent domain or pursue expropriation proceedings over a particular private property. 2. The power of eminent domain is exercised for public use, purpose or welfare, or for the benefit of the poor and the landless. 3. There is payment of just compensation, as required under Section 9, Article III of the Constitution, and other pertinent laws.

4. A valid and definite offer has been previously made to the owner of the property sought to be expropriated, but said offer was not accepted. In the case at bar, the local chief executive sought to exercise the power of eminent domain pursuant to a resolution of the municipal council. Thus, there was no compliance with the first requisite that the mayor be authorized through an ordinance. Petitioner cites Camarines Sur vs. CA to show that a resolution may suffice to support the exercise of eminent domain by an LGU. This case, however, is not in point because the applicable law at that time was BP 337, 30 the previous Local Government Code, which had provided that a mere resolution would enable an LGU to exercise eminent domain. In contrast, RA 7160 explicitly required an ordinance for this purpose. We are not convinced by petitioner's insistence that the terms "resolution" and "ordinance" are synonymous. A municipal ordinance is different from a resolution. An ordinance is a law, but a resolution is merely a declaration of the sentiment or opinion of a lawmaking body on a specific matter. An ordinance possesses a general and permanent character, but a resolution is temporary in nature. Additionally, the two are enacted differently - a third reading is necessary for an ordinance, but not for a resolution, unless decided otherwise by a majority of all the Sanggunian members. If Congress intended to allow LGUs to exercise eminent domain through a mere resolution, it would have simply adopted the language of the previous Local Government Code. But Congress did not. In a clear divergence from the previous Local Government Code, Section 19 of RA 7160 categorically requires that the local chief executive act pursuant to an ordinance. Indeed, "[l]egislative intent is determined principally from the language of a statute. Where the language of a statute is clear and unambiguous, the law is applied according to its express terms, and interpretation would be resorted to only where a literal interpretation would be either impossible or absurd or would lead to an injustice." 34 In the instant case, there is no reason to depart from this rule, since the law requiring an ordinance is not at all impossible, absurd, or unjust. Moreover, the power of eminent domain necessarily involves a derogation of a fundamental or private right of the people. 35 Accordingly, the manifest change in the legislative language - from "resolution" under the BP 337 to "ordinance" under RA 7160 - demands a strict construction. "No species of property is held by individuals with greater tenacity, and is guarded by the Constitution and laws more sedulously, than the right to the freehold of inhabitants. When the legislature interferes with that right and, for greater public purposes, appropriates the land of an individual without his consent, the plain meaning of the law should not be enlarged by doubtful interpretation." Petitioner relies on Article 36, Rule VI of the Implementing Rules, which requires only a resolution to authorize an LGU to exercise eminent domain. This is clearly misplaced, because Section 19 of RA 7160, the law itself, surely prevails over said rule which merely seeks to implement it. It is axiomatic that the clear letter of the law is controlling and cannot be amended by a mere administrative rule issued for its implementation. Besides, what the discrepancy seems to indicate is a mere oversight in the wording of the implementing rules, since Article 32, Rule VI thereof, also requires that, in exercising the power of eminent domain, the chief executive of the LGU must act pursuant to an ordinance. Issue: WON the complaint states a cause of action Held: No Ratio: In the first place, petitioner merely alleged the existence of such an ordinance, but it did not present any certified true copy thereof. In the second place, petitioner did not raise this point before this Court. In fact, it was mentioned by private respondent, and only in passing. In any event, this allegation does not cure the inherent defect of petitioner's Complaint for expropriation filed on September 23, 1993. The fact that there is no cause of action is evident from the face of the Complaint for expropriation which was based on a mere resolution. The absence of an ordinance authorizing the same is equivalent to lack of cause of action. Consequently, the Court of Appeals committed no reversible error in affirming the trial court's Decision which dismissed the expropriation suit.

Issue: WON the action is bared by res juridicata Held: No

Ratio: All the requisites for the application of res judicata are present in this case. There is a previous final judgment on the merits in a prior expropriation case involving identical interests, subject matter and cause of action, which has been rendered by a court having jurisdiction over it. Be that as it may, the Court holds that the principle of res judicata, which finds application in generally all cases and proceedings, cannot bar the right of the State or its agent to expropriate private property. The very nature of eminent domain, as an inherent power of the State, dictates that the right to exercise the power be absolute and unfettered even by a prior judgment or res judicata. The scope of eminent domain is plenary and, like police power, can "reach every form of property which the State might need for public use." "All separate interests of individuals in property are held of the government under this tacit agreement or implied reservation. Notwithstanding the grant to individuals, the eminent domain, the highest and most exact idea of property, remains in the government, or in the aggregate body of the people in their sovereign capacity; and they have the right to resume the possession of the property whenever the public interest requires it." Thus, the State or its authorized agent cannot be forever barred from exercising said right by reason alone of previous non-compliance with any legal requirement. While the principle of res judicata does not denigrate the right of the State to exercise eminent domain, it does apply to specific issues decided in a previous case. For example, a final judgment dismissing an expropriation suit on the ground that there was no prior offer precludes another suit raising the same issue; it cannot, however, bar the State or its agent from thereafter complying with this requirement, as prescribed by law, and subsequently exercising its power of eminent domain over the same property. By the same token, our ruling that petitioner cannot exercise its delegated power of eminent domain through a mere resolution will not bar it from reinstituting similar proceedings, once the said legal requirement and, for that matter, all others are properly complied with. Parenthetically and by parity of reasoning, the same is also true of the principle of "law of the case." In Republic vs De Knecht, the Court ruled that the power of the State or its agent to exercise eminent domain is not diminished by the mere fact that a prior final judgment over the property to be expropriated has become the law of the case as to the parties. The State or its authorized agent may still subsequently exercise its right to expropriate the same property, once all legal requirements are complied with. To rule otherwise will not only improperly diminish the power of eminent domain, but also clearly defeat social justice. City of Cebu vs. CA Facts: Merlita Cardeno is the owner of a parcel of land in Sitio Sto. Nino, Alaska-Mambaling. The City of Cebu, filed a complaint for eminent domain against Cardeno with the RTC seeking to expropriate the said parcel of land. The complaint was initiated pursuant to Resolution No. 404 and Ordinance No. 1418, dated February 17, 1992, of the Sangguniang Panlungsod of Cebu City authorizing the City Mayor to expropriate the said parcel of land for the purpose of providing a socialized housing project for the landless and low-income city residents. Cardeno filed a motion to dismiss on the ground of lack of cause of action as there has been negotiations for the purchase of the property without resorting to expropriation, but said negotiations failed. Also, there was no compliance with the conditions to the exercise of the power of eminent domain (valid and definite offer made to the owner and non acceptance). The RTC dismissed the complaint. The CA affirmed the of the RTC. According to the CA, an allegation of repeated negotiations made with the private respondent for the purchase of her property by the petitioner, "cannot by any stretch of imagination, be equated or likened to the clear and specific requirement that the petitioner should have previously made a valid and definite offer to purchase." It further added that the term "negotiation" which necessarily implies uncertainty, it consisting of acts the purpose of which is to arrive at a conclusion, may not be perceived to mean the valid and definite offer contemplated by law. Issue: WON expropriation may be granted

Ratio: An offshoot of the foregoing is the instant petition for review on certiorari which has essentially become a battle of semantics being waged before this Court. While petitioner reiterates that paragraph VII of the complaint sufficiently states compliance with the requirement of "a valid and definite offer", private respondent insists that the term "negotiations" is too broad to be equated with the said requirement. Elaborating, private respondent posited that by definition, "negotiations run the whole range of acts preparatory to concluding an agreement, from the preliminary correspondence; the fixing of the terms of the agreement; the price; the mode of payment; obligations of (sic) the parties may conceive as necessary to their agreement." Thus, "negotiations" by itself may pertain to any of the foregoing and does not automatically mean the making of "a valid and definite offer." At the outset, it must be said that without necessarily delving into the parties' semantical arguments, this Court finds that the complaint does in fact state a cause of action. What may perhaps be conceded is only the relative ambiguity of the allegations in paragraph VII of the complaint. However, as We have previously held, a complaint should not be dismissed upon a mere ambiguity, indefiniteness or uncertainty of the cause of action stated thereinfor these are not grounds for a motion to dismiss but rather for a bill of particulars. In other words, a complaint should not be dismissed for insufficiency unless it appears clearly from the face of the complaint that the plaintiff is not entitled to any relief under any state of facts which could be proved within the facts alleged therein. The error of both the RTC and respondent Court of Appeals in holding that the complaint failed to state a cause of action stems from their inflexible application of the rule that: when the motion to dismiss is based on the ground that the complaint states no cause of action, no evidence may be allowed and the issue should only be determined in the light of the allegations of the complaint. However, this rule is not without exceptions. In the case of Tan v. Director of Forestry, this Court departed from the aforementioned rule and held that, ". . . although the evidence of the parties were on the question of granting or denying the petitioner-appellant's application for a writ of preliminary injunction, the trial court correctly applied said evidence in the resolution of the motion to dismiss." Likewise, in Marcopper Mining Corporation v. Garcia, we sanctioned the act of the trial court in considering, in addition to the complaint, other pleadings submitted by the parties in deciding whether or not the complaint should be dismissed for lack of cause of action. This Court deemed such course of action but logical where the trial court had the opportunity to examine the merits of the complaint, the answer with counterclaim, the petitioner's answer to the counterclaim and its answer to the request for admission. The same liberality should be applied in the instant case. Furthermore, even on the face of the complaint alone, there is extant a cause of action. All documents attached to a complaint, the due execution and genuineness of which are not detained under oath by the defendant, must be considered as part of the complaint without need of introducing evidence thereon. Additionally, the general rule is that a motion to dismiss hypothetically admits the truth of the facts alleged in the complaint. Thus, Ordinance No. 1418 is not only incorporated into the complaint for eminent domain filed by petitioner, but is also deemed admitted by private respondent. A perusal of the copy of said ordinance which has been annexed to the complaint shows that the fact of petitioner's having made a previous valid and definite offer to private respondent is categorically stated therein. Thus, the second whereas clause of the said ordinance provides as follows: WHEREAS, the city government has made a valid and definite offer to purchase subject lot(s) for the public use aforementioned but the registered owner Mrs. Merlita Cardeno has rejected such offer. The foregoing should now put to rest the long drawn argument over the alleged failure of the complaint to state a cause of action. There is no longer any room for doubt that as alleged in the complaint, and as admitted by private respondent, the petitioner had in fact complied with the condition precedent of "a valid and definite offer" set forth in Sec. 19 of R.A. 7160. The rules of procedure are not to be applied in a very rigid, technical sense; rules of procedure are used only to help secure substantial justice. If a technical and rigid enforcement of the rules is made their aim would be defeated. Where the rules are merely secondary in

importance are made to override the ends of justice; the technical rules had been misapplied to the prejudice of the substantial right of a party, said rigid application cannot be countenanced. The doctrine finds compelling application in the case at bench. For as correctly averred by petitioner, nothing else was accomplished by the dismissal of the complaint for eminent domain but a considerable delay in the proceedings. The dismissal of the complaint did not bar petitioner from filing another eminent domain case and from correcting its alleged error by the mere expedient of changing paragraph VII thereof. Indeed, precious time has been wasted while the salutary objectives of Ordinance No. 1418 of the City of Cebu have been put on hold by a quarrel over technical matters. Francia vs. Mun. of Meycauyan Facts: On February 6, 2003, the respondent filed a complaint for expropriation against petitioners. Respondent needed petitioners' 16,256 sq. m. idle property at the junction of the North Expressway, Malhacan-Iba-Camalig main road artery and the MacArthur Highway. It planned to use it to establish a common public terminal for all types of public utility vehicles with a weighing scale for heavy trucks. In their answer, petitioners denied that the property sought to be expropriated was raw land. It was in fact developed and there were plans for further development. For this reason, respondent's offer price of P2,333,500 (or P111.99 per square meter) was too low. After trial, the RTC ruled that the expropriation was for a public purpose as the terminal would improve the flow of traffic during rush hours. Moreover, the property as the best site for the proposed terminal because of its accessibility. Aggrieved, petitioners filed a petition for certiorari in the CA. They claimed that the trial court issued the orders without conducting a hearing to determine the existence of a public purpose. The partially granted the petition. Finding that petitioners were deprived of an opportunity to controvert respondent's allegations, the appellate court nullified the order of expropriation except with regard to the writ of possession. According to the CA, a hearing was not necessary because once the expropriator deposited the required amount (with the Court), the issuance of a writ of possession became ministerial. Issue: WON the CA erred in upholding the RTC's orders that, in expropriation cases, prior determination of the existence of a public purpose was not necessary for the issuance of a writ of possession. Held: No Ratio: Section 19 of Republic Act 7160[9] provides: Section 19. Eminent Domain. A local government unit may, through its chief executive and acting pursuant to an ordinance, exercise the power of eminent domain for public use, or purpose, or welfare for the benefit of the poor and the landless, upon payment of just compensation, pursuant to the provisions of the Constitution and pertinent laws; Provided, however, That the power of eminent domain may not be exercised unless a valid and definite offer has been previously made to the owner, and that such offer was not accepted; Provided, further, That the local government unit may immediately take possession of the property upon the filing of the expropriation proceedings and upon making a deposit with the proper court of at least fifteen percent (15%) of the fair market value of the property based on the current tax declaration of the property to be expropriated; Provided, finally, That, the amount to be paid for the expropriated property shall be determined by the proper court, based on the fair market value at the time of the taking of the property. Before a local government unit may enter into the possession of the property sought to be expropriated, it must (1) file a complaint for expropriation sufficient in form and substance in the proper court and (2) deposit with the said court at least 15% of the property's fair market value based on its current tax declaration. The law does not make the determination of a public purpose a condition precedent to the issuance of a writ of possession. Heirs of Ardona v. Reyes Facts: The Philippine Tourism Authority filed 4 complaints with the CFI of Cebu City for the expropriation of 282 ha of rolling land situated in barangays Malubog and Babag, Cebu City for the development into integrated resort complexes of selected and well-defined geographic areas with potential tourism value. The

PTA will construct a sports complex, club house, golf course, playground and picnic area on said land. An electric power grid will also be established by NPC as well as deep well and drainage system. Complimentary support facilities (malls, coffee shops, etc) will also be created. The defendants alleged that the taking is allegedly not impressed with public use under the Constitution. Also, assuming that PTA has such power, the intended use cannot be paramount to the determination of the land as a land reform area; that limiting the amount of compensation by legislative fiat is constitutionally repugnant; and that since the land is under the land reform program, it is the Court of Agrarian Relations and not the Court of First Instance, that has jurisdiction over the expropriation cases. The Philippine Tourism Authority having deposited with the PNB, an amount equivalent to 10% of the value of the properties pursuant to PD1533, the lower court issued separate orders authorizing PTA to take immediate possession of the premises and directing the issuance of writs of possession. Issue: WON the public use requirement has been complied with Held: Yes Ratio: There are three provisions of the Constitution which directly provide for the exercise of the power of eminent domain. Sec 2, Article IV states that private property shall not be taken for public use without just compensation. Section 6, Article XIV allows the State, in the interest of national welfare or defense and upon payment of just compensation to transfer to public ownership, utilities and other private enterprises to be operated by the government. Section 13, Article XIV states that the Batasang Pambansa may authorize upon payment of just compensation the expropriation of private lands to be subdivided into small lots and conveyed at cost to deserving citizens. While not directly mentioning the expropriation of private properties upon payment of just compensation, the provisions on social justice and agrarian reforms which allow the exercise of police power together with the power of eminent domain in the implementation of constitutional objectives are even more far reaching insofar as taxing of private property is concerned. We cite all the above provisions on the power to expropriate because of the petitioners' insistence on a restrictive view of the eminent domain provision. The thrust of all constitutional provisions on expropriation is in the opposite direction. As early as 1919, this Court in Visayan Refining Co. v. Samus categorized the restrictive view as wholly erroneous and based on a misconception of fundamentals. The petitioners look for the word "tourism" in the Constitution. Understandably the search would be in vain. To freeze specific programs like tourism into express constitutional provisions would make the Constitution more prolix than a bulky code and require of the framers a prescience beyond Delphic proportions. In said case, this Court emphasized that the power of eminent domain is inseparable from sovereignty being essential to the existence of the State and inherent in government even in its most primitive forms. The only purpose of the provision in the Bill of Rights is to provide some form of restraint on the sovereign power. It is not a grant of authority . The petitioners ask us to adopt a strict construction and declare that "public use" means literally use by the public and that "public use" is not synonymous with "public interest", "public benefit", or "public welfare" and much less "public convenience." The petitioners face two major obstacles. First, their contention which is rather sweeping in its call for a retreat from the public welfare orientation is unduly restrictive and outmoded. Second, no less than the lawmaker has made a policy determination that the power of eminent domain may be exercised in the promotion and development of Philippine tourism. The restrictive view of public use may be appropriate for a nation which circumscribes the scope of government activities and public concerns and which possesses big and correctly located public lands that obviate the need to take private property for public purposes. Neither circumstance applies to the Philippines. We have never been a laissez faire State. And the necessities which impel the exertion of sovereign power are all too often found in areas of scarce public land or limited government resources. There can be no doubt that expropriation for such traditional purposes as the construction of roads, bridges, ports, waterworks, schools, electric and telecommunications systems, hydroelectric power plants, markets

and slaughterhouses, parks, hospitals, government office buildings, and flood control systems is valid. However, the concept of public use is not limited to traditional purposes. Here as elsewhere the idea that "public use" is strictly limited to clear cases of "use by the public" has been discarded. In the Philippines, Chief Justice Enrique M. Fernando has aptly summarized the statutory and judicial trend as follows: "The taking to be valid must be for public use. There was a time when it was felt that a literal meaning should be attached to such a requirement. Whatever project is undertaken must be for the public to enjoy, as in the case of streets or parks. Otherwise, expropriation is not allowable. It is not any more. As long as the purpose of the taking is public, then the power of eminent domain comes into play. As just noted, the constitution in at least two cases, to remove any doubt, determines what is public use. One is the expropriation of lands to be subdivided into small lots for resale at cost to individuals. The other is in the transfer, through the exercise of this power, of utilities and other private enterprise to the government. It is accurate to state then that at present whatever may be beneficially employed for the general welfare satisfies the requirement of public use." The petitioners' contention that the promotion of tourism is not "public use" because private concessioners would be allowed to maintain various facilities such as restaurants, hotels, stores, etc. inside the tourist complex is impressed with even less merit. Private bus firms, taxicab fleets, roadside restaurants, and other private businesses using public streets and highways do not diminish in the least bit the public character of expropriations for roads and streets. The lease of store spaces in underpasses of streets built on expropriated land does not make the taking for a private purpose. Airports and piers catering exclusively to private airlines and shipping companies are still for public use. The expropriation of private land for slum clearance and urban development is for a public purpose even if the developed area is later sold to private homeowners, commercial firms, entertainment and service companies, and other private concerns. The petitioners have also failed to overcome the deference that is appropriately accorded to formulations of national policy expressed in legislation. The rule in Berman v. Parker (supra) of deference to legislative policy even if such policy might mean taking from one private person and conferring on another private person applies as well as in the Philippines. An examination of the language in the 1919 cases of City of Manila v. Chinese Community of Manila and Visayan Refining Co. v. Camus, earlier cited, shows that from the very start of constitutional government in our country judicial deference to legislative policy has been clear and manifest in eminent domain proceedings. The expressions of national policy are found in the revised charter of the Philippine Tourism Authority, PD 564. (Disregard of Land Reform Nature) According to them, assuming that PTA has the right to expropriate, the properties subject of expropriation may not be taken for the purposes intended since they are within the coverage of "operation land transfer" under the land reform program; that the agrarian reform program occupies a higher level in the order of priorities than other State policies like those relating to the health and physical well-being of the people; and that property already taken for public use may not be taken for another public use. The petitioners, however, have failed to show that the area being developed is indeed a land reform area and that the affected persons have emancipation patents and certificates of land transfer. The records show that the area being developed into a tourism complex consists of more than 808 hectares, almost all of which is not affected by the land reform program. The portion being expropriated is 282 hectares of hilly and unproductive land where even subsistence farming of crops other than rice and corn can hardly survive. And of the 282 disputed hectares, only 8,970 square meters - less than one hectare - is affected by Operation Land Transfer. Of the 40 defendants, only two have emancipation patents for the less than one hectare of land affected. (Non Impairment Clause) The non-impairment clause has never been a barrier to the exercise of police power and likewise eminent domain. As stated in Manigault v. Springs "parties by entering into contracts may not estop the legislature from enacting laws intended for the public good." The applicable doctrine is expressed in Arce v. Genato which involved the expropriation of land for a public plaza. The issue of prematurity is also raised by the petitioners. They claim that since the necessity for the taking has not been

previously established, the issuance of the orders authorizing the PTA to take immediate possession of the premises, as well as the corresponding writs of possession was premature. Under Presidential Decree No. 42, as amended by Presidential Decree No. 1533, the government, its agency or instrumentality, as plaintiff in an expropriation proceedings is authorized to take immediate possession, control and disposition of the property and the improvements, with power of demolition, notwithstanding the pendency of the issues before the court, upon deposit with the Philippine National Bank of an amount equivalent to 10% of the value of the property expropriated. The issue of immediate possession has been settled in Arce v. Genato. In answer to the issue: ". . . condemnation or expropriation proceedings is in the nature of one that is quasi-in-rem, wherein the fact that the owner of the property is made a party is not essentially indispensable insofar at least as it concerns the immediate taking of possession of the property and the preliminary determination of its value, including the amount to be deposited." Makasiar: It appearing that the petitioners are not tenants of the parcels of land in question and therefore do not fall within the purview of the Land Reform Code, the petition should be dismissed on that score alone. There is no need to decide whether the power of the PTA to expropriate the land in question predicated on the police power of the State shall take precedence over the social justice guarantee in favor of tenants and the landless. The welfare of the landless and small land owners should prevail over the right of the PTA to expropriate the lands just to develop tourism industry, which benefit the wealthy only. Such a position would increase the disenchanted citizens and drive them to dissidence. The government is instituted primarily for the welfare of the governed and there are more poor people in this country than the rich. The tourism industry is not essential to the existence of the government, but the citizens are, and their right to live in dignity should take precedence over the development of the tourism industry. Filstream International Inc. vs. CA Facts: Petitioner is the registered owner of the properties subject of this dispute consisting of adjacent parcels of land situated in Antonio Rivera Street, Tondo II, Manila. Petitioner filed an ejectment suit before the MTC against the occupants of the parcels of land (private respondents) on the grounds of termination of the lease contract and non-payment of rentals. Judgment was rendered by the MTC ordering private respondents to vacate the premises and pay back rentals to petitioner. The RTC and CA affirmed. However, it appeared that during the pendency of the ejectment proceedings private respondents, a complaint for Annulment of Deed of Exchange against petitioner which was filed before the RTC. The City of Manila then approved Ordinance No. 7813, authorizing Mayor Lim to initiate the acquisition by negotiation, expropriation, purchase, or other legal means certain parcels of land which formed part of the properties of petitioner then occupied by private respondents. The City approved Ordinance No. 7855 declaring the expropriation of certain parcels of land situated along Antonio Rivera and Fernando Ma. Guerero streets in Tondo, Manila which were owned by Mr. Enrique Quijano Gutierez, petitioners predecessor -in-interest. The said properties were to be sold and distributed to qualified tenants of the area pursuant to the Land Use Development Program of the City of Manila. The City of Manila filed a complaint for eminent domain. The trial court issued a Writ of Possession in favor of the city which ordered the transfer of possession over the disputed premises to the City of Manila. Petitioner filed a motion to dismiss but the court denied th emotion. Concerning the first case, the trial court issued an order commanding the demolition of the structure erected on the disputed premises. To avert the demolition, private respondents filed before the RTC of Manila, Branch 14, a Petition for Certiorari and Prohibition with prayer for the issuance of a temporary restraining order and preliminary injunction. The TRO was granted which was later lifted. The court the dismissed the case on the ground of forum shopping. On appeal, the CA found merit in private respondents allegations in support of their application of the issuance of the writ and granted the same. The issue raised in G.R. No. 125218 is purely procedural and technical matter. Petitioner takes exception to the resolutions of the CA which ordered the dismissal of its Petition for Certiorari for non-compliance with Sec. 2(a) of Rule 6 of the Revised Internal Rules of the Court of Appeals by failing to attach to its petition other

pertinent documents and papers and for attaching copies of pleadings which are blurred and unreadable. Petitioner argues that respondent appellate court seriously erred in giving more premium to form rather than the substance. We agree with the petitioner. A strict adherence to the technical and procedural rules in this case would defeat rather than meet the ends of justice as it would result in the violation of the substantial rights of petitioner. At stake in the appeal filed by petitioner before the CA is the exercise of their property rights over the disputed premises which have been expropriated and have in fact been ordered condemned in favor of the City of Manila. In effect, the dismissal of their appeal in the expropriation proceedings based on the aforementioned grounds is tantamount to a deprivation of property without due process of law as it would automatically validate the expropriation proceedings based on the aforementioned grounds is tantamount to a deprivation of property without due process of law as it would automatically validate the expropriation proceedings which the petitioner is still disputing. It must be emphasized that where substantial rights are affected, as in this case, the stringent application of procedural rules may be relaxed if only to meet the ends of substantial justice. With regard to the other petition, G.R. No. 128077, petitioner Filstream objects to the issuance by respondent CA of the restraining order and the preliminary injunction enjoining the execution of the writ of demolition issued in the ejectment suit as an incident to private respondents pending petition assailing the dismissal by the RTC of Manila, Branch 33, of the consolidated petitions for certiorari filed by private respondents and the City of Manila on the ground of forum shopping. The propriety of the issuance of the restraining order and the writ of preliminary injunction is but a mere incient to the actual controversy which is rooted in the assertion of the conflicting rights of the parties in this case over the disputed premises. In order to determine whether private respondents are entitled to the injunctive reliefs granted by respondent CA, we deemed it proper to extract the source of discord. Petitioner anchors its claim by virtue of its ownership over the properties and the existence of a final and executory judgment against private respondents ordering the latters ejectment from the premises. Private respondents claim on the other hand hinges on an alleged supervening event which has rendered the enforcement of petitioners rights moot, that is, the expropriation proceedings undertaken by the City of Manila over the disputed premises for the benefit of herein private respondents. For its part, the City is merely exercising its power of eminent domain within its jurisdiction by expropriating petitioners properties for public use. There is no dispute as to the existence of a final and executory judgment in favor of petitioner ordering the ejectment of private respondents from the properties subject of this dispute. The judgment in the ejectment suit became final after private respondents failed to interpose any appeal from the adverse decision of CA. Petitioner has every right to assert the execution of this decision as it had already became final and executory. However, it must also be conceded that the City of Manila has an undeniable right to exercise its power of eminent domain within its jurisdiction. The right to expropriate private property for public use is expressly granted to it under Section 19 of the 1991 Local Government Code. More specifically, the City of Manila has the power to expropriate private property in the pursuit of its urban land reform and housing program as explicitly laid out in the Revised Charter of the City of Manila (R.A. No. 409). In fact, the City of Manilas right to exercise these prerogatives notwithstanding the existence of a final and executory judgment over the property to be expropriated has been upheld by this Court in the case of Philippine Columbian Association vs. Panis. Corollary to the expanded notion of public use, expropriation is not anymore confined to vast tracts of land and landed estate. It is therefore of no moment that the land sought to be expropriated in this case is less than the half a hectare only.

Through the years, the public use requirement in eminent domain has evolved into a flexible concept, influenced by changing conditions. Public use now includes the broader notion of indirect public benefit or advantage, including a particular, urban land reform and housing. We take judicial notice of the fact that urban land reform has become a paramount task in view of the acute shortage of decent housing in urban areas. Nevertheless, despite the existence of a serious dilemma, local government units are not given an unbridled authority when exercising their power of eminent domain in pursuit of solutions to these problems. The basic rules still have to be followed, which are as follows: no person shall be deprived of life, liberty, or property without due process of law, nor shall any person be denied the equal protection of the laws; private property shall not be taken for public use without just compensation. Thus the exercise by local government units of the power of eminent domain is not without limitations. Where on-site development is found more practicable and advantageous to the beneficiaries, the priorities mentioned in this section shall not apply. The local government units shall give budgetary priority to on-site development of government lands. Very clear from the provisions are the limitations with respect to the order of priority in acquiring private lands and in resorting to expropriation proceedings as means to acquire the same. Private lands rank last in the order of priority for purposes of socialized housing. In the same vein, expropriation proceedings are to be resorted to only when the other modes of acquisition have been exhausted. Compliance with these conditions must be deemed mandatory because these are the only safeguards in securing the right of owners of private property to due process when their property is expropriated for public use. Proceeding from the parameters laid out in the above disquisitions, we now pose the crucial question: Did the city of Manila comply with the abovementioned conditions when it expropriated petitioners properties? We have carefully scrutinized the records of this case and found nothing that would indicate the respondent City of Manila complied with Sec. 9 and Sec. 10 of R.A. 7279. Petitioners properties were expropriated and ordered condemned in favor of the City of Manila sans any showing that resort to the acquisition of other lands listed under Sec. 9 of RA 7279 have proved futile. Evidently, there was a violation of petitioner Filstreams right to due process which must accordingly be rectified. Indeed, it must be emphasized that the State has a paramount interest in exercising its power of eminent domain for the general good considering that the right of the State to expropriate private property as long as it is for public use always takes precedence over the interest of private property owners. However we must not lose sight of the fact that the individual rights affected by the exercise of such right are also entitled to protection, bearing in mind that the exercise of this superior right cannot override the guarantee of due process extended by the law to owners of the property to be expropriated. In this regard, vigilance over compliance with the due process requirements is in order. Hagonoy Market Vendors Assn. Vs. Mun. of Hagonoy, Bulacan Facts: On October 1, 1996, the Sangguniang Bayan of Hagonoy, Bulacan, enacted an ordinance, Kautusan Blg. 28, which increased the stall rentals of the market vendors in Hagonoy. Article 3 provided that it shall take effect upon approval. The subject ordinance was posted from November 4-25, 1996. In the last week of November, 1997, the petitioners members were personally given copies of the approved Ordinance and were informed that it shall be enforced in January, 1998. On December 8, 1997, the petitioners President filed an appeal with the Secretary of Justice assailing the constitutionality of the tax ordinance. Petitioner claimed it was unaware of the posting of the ordinance. Respondent opposed the appeal. It contended that the ordinance took effect on October 6, 1996 and that the ordinance, as approved, was posted as required by law. Hence, it was pointed out that petitioners appeal, made over a year later, was already time-barred.

The Secretary of Justice dismissed the appeal on the ground that it was filed out of time, i.e., beyond thirty (30) days from the effectivity of the Ordinance on October 1, 1996, as prescribed under Section 187 of the 1991 Local Government Code Issue: WON the CA erred in dismissing the appeal Held: Yes Ratio: We find that the Court of Appeals erred in dismissing peti tioners appeal on the ground that it was formally deficient. It is clear from the records that the petitioner exerted due diligence to get the copies of its appealed Resolutions certified by the Department of Justice, but failed to do so on account of typhoon Loleng. Under the circumstances, respondent appellate court should have tempered its strict application of procedural rules in view of the fortuitous event considering that litigation is not a game of technicalities. Issue: WON the petitioner should be dismissed Held: Yes Ratio: The petition should be dismissed as the appeal of the petitioner with the Secretary of Justice is already time-barred. The applicable law is Section 187 of the 1991 Local Government Code. The law requires that an appeal of a tax ordinance or revenue measure should be made to the Secretary of Justice within thirty (30) days from effectivity of the ordinance and even during its pendency, the effectivity of the assailed ordinance shall not be suspended. In the case at bar, Municipal Ordinance No. 28 took effect in October 1996. Petitioner filed its appeal only in December 1997, more than a year after the effectivity of the ordinance in 1996. Clearly, the Secretary of Justice correctly dismissed it for being time-barred. At this point, it is apropos to state that the timeframe fixed by law for parties to avail of their legal remedies before competent courts is not a mere technicality that can be easily brushed aside. The periods stated in Section 187 of the Local Government Code are mandatory.[10] Ordinance No. 28 is a revenue measure adopted by the municipality of Hagonoy to fix and collect public market stall rentals. Being its lifeblood, collection of revenues by the government is of paramount importance. The funds for the operation of its agencies and provision of basic services to its inhabitants are largely derived from its revenues and collections. Thus, it is essential that the validity of revenue measures is not left uncertain for a considerable length of time. Hence, the law provided a time limit for an aggrieved party to assail the legality of revenue measures and tax ordinances. In a last ditch effort to justify its failure to file a timely appeal with the Secretary of Justice, the petitioner contends that its period to appeal should be counted not from the time the ordinance took effect in 1996 but from the time its members were personally given copies of the approved ordinance in November 1997. It insists that it was unaware of the approval and effectivity of the subject ordinance in 1996 on two (2) grounds: first, no public hearing was conducted prior to the passage of the ordinance and, second, the approved ordinance was not posted. We do not agree. Petitioners bold assertion that there was no public hearing conducted prior to the passage of Kautusan Blg. 28 is belied by its own evidence. In petitioners two (2) communications with the Secretary of Justice,[12] it enumerated the various objections raised by its members before the passage of the ordinance in several meetings called by the Sanggunian for the purpose. These show beyond doubt that petitioner was aware of the proposed increase and in fact participated in the public hearings therefor. The respondent municipality likewise submitted the Minutes and Report of the public hearings conducted by the Sangguniang Bayans Committee on Appropriations and Market on February 6, July 15 and August 19, all in 1996, for the proposed increase in the stall rentals. Petitioner cannot gripe that there was practically no public hearing conducted as its objections to the proposed measure were not considered by the Sangguniang Bayan. To be sure, public hearings are conducted by legislative bodies to allow interested parties to ventilate their views on a proposed law or ordinance. These views, however, are not binding on the legislative body and it is not compelled by law to adopt the same. Sanggunian members are elected by the people to make laws that will promote the general interest of

their constituents. They are mandated to use their discretion and best judgment in serving the people. Parties who participate in public hearings to give their opinions on a proposed ordinance should not expect that their views would be patronized by their lawmakers. On the issue of publication or posting, (Section 188 of the Local Government Code), the records is bereft of any evidence to prove petitioners negative allegation that the subject ordinance was not posted as required by law. In contrast, the respondent Sangguniang Bayan of the Municipality of Hagonoy, Bulacan, presented evidence which clearly shows that the procedure for the enactment of the assailed ordinance was complied with. Municipal Ordinance No. 28 was enacted by the Sangguniang Bayan of Hagonoy on October 1, 1996. Then Acting Municipal Mayor Maria Garcia Santos approved the Ordinance on October 7, 1996. After its approval, copies of the Ordinance were given to the Municipal Treasurer on the same day. On November 9, 1996, the Ordinance was approved by the Sangguniang Panlalawigan. The Ordinance was posted during the period from November 4 - 25, 1996 in three (3) public places, viz: in front of the municipal building, at the bulletin board of the Sta. Ana Parish Church and on the front door of the Office of the Market Master in the public market.[14] Posting was validly made in lieu of publication as there was no newspaper of local circulation in the municipality of Hagonoy. This fact was known to and admitted by petitioner. Thus, petitioners ambiguous and unsupported claim that it was only sometime in November 1997 that the Provincial Board approved Municipal Ordinance No. 28 and so the posting could not have been made in November 1996 was sufficiently disproved by the positive evidence of respondent municipality. Given the foregoing circumstances, petitioner cannot validly claim lack of knowledge of the approved ordinance. The filing of its appeal a year after the effectivity of the subject ordinance is fatal to its cause. Finally, even on the substantive points raised, the petition must fail. Section 6c.04 of the 1993 Municipal Revenue Code and Section 191 of the Local Government Code limiting the percentage of increase that can be imposed apply to tax rates, not rentals. Neither can it be said that the rates were not uniformly imposed or that the public markets included in the Ordinance were unreasonably determined or classified. To be sure, the Ordinance covered the three (3) concrete public markets: the two-storey Bagong Palengke, the burnt but reconstructed Lumang Palengke and the more recent Lumang Palengke with wet market. However, the Palengkeng Bagong Munisipyo or Gabaldon was excluded from the increase in rentals as it is only a makeshift, dilapidated place, with no doors or protection for security, intended for transient peddlers who used to sell their goods along the sidewalk. Republic of the Philippines vs. Court of Appeals Facts: Petitioner instituted expropriation proceedingscovering a total of 544,980 square meters of contiguous land situated along MacArthur Highway, Malolos, Bulacan, to be utilized for the continued broadcast operation and use of radio transmitter facilities for the Voice of the Philippines project. Petitioner took over the premises after the previous lessee, the Voice of America, had ceased its operations thereat. Petitioner made a deposit of P517,558.80, the sum provisionally fixed as being the reasonable value of the property. On 26 February 1979, or more than nine years after the institution of the expropriation proceedings, the trial court issued this order condemning the property and ordering the plaintiff to pay the defendants the just compensation for the property. The bone of contention is the 76,589-square meter property previously owned by Luis Santos, predecessorin-interest of respondents, which forms part of the expropriated area. It appears that the national government failed to pay to respondents the compensation pursuant to the foregoing decision, such that a little over five years later, or on 09 May 1984, respondents filed a manifestation with a motion seeking payment for the expropriated property. In the meantime, President Estrada issued Proclamation No. 22, transferring 20 hectares of the expropriated property to the Bulacan State University for the expansion of its facilities and another 5 hectares to be used exclusively for the propagation of the Philippine carabao. The remaining portion was retained by the PIA. This fact notwithstanding, and despite the 1984 court order, the Santos heirs remained unpaid, and no action was taken on their case until 16 September 1999 when petitioner filed its manifestation and motion to permit the deposit in court of the amount of P4,664,000.00 by way of just compensation for the expropriated

property of the late Luis Santos subject to such final computation as might be approved by the court. This time, the Santos heirs, opposing the manifestation and motion, submitted a counter-motion to adjust the compensation from P6.00 per square meter previously fixed in the 1979 decision to its current zonal valuation pegged at P5,000.00 per square meter or, in the alternative, to cause the return to them of the expropriated property. On 01 March 2000, the Bulacan RTC ruled in favor of respondents and issued the assailed order, vacating its decision of 26 February 1979 and declaring it to be unenforceable on the ground of prescription. The CA denied the appeal (failure to file during the reglementary period). The right of eminent domain is usually understood to be an ultimate right of the sovereign power to appropriate any property within its territorial sovereignty for a public purpose. Fundamental to the independent existence of a State, it requires no recognition by the Constitution, whose provisions are taken as being merely confirmatory of its presence and as being regulatory, at most, in the due exercise of the power. In the hands of the legislature, the power is inherent, its scope matching that of taxation, even that of police power itself, in many respects. It reaches to every form of property the State needs for public use and, as an old case so puts it, all separate interests of individuals in property are held under a tacit agreement or implied reservation vesting upon the sovereign the right to resume the possession of the property whenever the public interest so requires it The ubiquitous character of eminent domain is manifest in the nature of the expropriation proceedings. Expropriation proceedings are not adversarial in the conventional sense, for the condemning authority is not required to assert any conflicting interest in the property. Thus, by filing the action, the condemnor in effect merely serves notice that it is taking title and possession of the property, and the defendant asserts title or interest in the property, not to prove a right to possession, but to prove a right to compensation for the taking. Obviously, however, the power is not without its limits: first, the taking must be for public use, and second, that just compensation must be given to the private owner of the property. These twin proscriptions have their origin in the recognition of the necessity for achieving balance between the State interests, on the one hand, and private rights, upon the other hand, by effectively restraining the former and affording protection to the latter. In determining public use, two approaches are utilized - the first is public employment or the actual use by the public, and the second is public advantage or benefit. It is also useful to view the matter as being subject to constant growth, which is to say that as society advances, its demands upon the individual so increases, and each demand is a new use to which the resources of the individual may be devoted. The expropriated property has been shown to be for the continued utilization by the PIA, a significant portion thereof being ceded for the expansion of the facilities of the Bulacan State University and for the propagation of the Philippine carabao, themselves in line with the requirements of public purpose. Respondents question the public nature of the utilization by petitioner of the condemned property, pointing out that its present use differs from the purpose originally contemplated in the 1969 expropriation proceedings. The argument is of no moment. The property has assumed a public character upon its expropriation. Surely, petitioner, as the condemnor and as the owner of the property, is well within its rights to alter and decide the use of that property, the only limitation being that it be for public use, which, decidedly, it is. In insisting on the return of the expropriated property, respondents would exhort on the pronouncement in Provincial Government of Sorsogon vs. Vda. de Villaroya where the unpaid landowners were allowed the alternative remedy of recovery of the property there in question. It might be borne in mind that the case involved the municipal government of Sorsogon, to which the power of eminent domain is not inherent, but merely delegated and of limited application. The grant of the power of eminent domain to local governments under RA 7160 cannot be understood as being the pervasive and all-encompassing power vested in the legislative branch of government. For local governments to be able to wield the power, it must, by enabling law, be delegated to it by the national legislature, but even then, this delegated power of eminent domain is not, strictly speaking, a power of eminent, but only of inferior, domain or only as broad or confined as the real authority would want it to be.

Thus, in Valdehueza vs. Republic[17] where the private landowners had remained unpaid ten years after the termination of the expropriation proceedings, this Court ruled - The points in dispute are whether such payment can still be made and, if so, in what amount. Said lots have been the subject of expropriation proceedings. By final and executory judgment in said proceedings, they were condemned for public use, as part of an airport, and ordered sold to the government. x x x It follows that both by virtue of the judgment, long final, in the expropriation suit, as well as the annotations upon their title certificates, plaintiffs are not entitled to recover possession of their expropriated lots - which are still devoted to the public use for which they were expropriated - but only to demand the fair market value of the same. The judgment rendered by the Bulacan RTC in 1979 on the expropriation proceedings provides not only for the payment of just compensation to herein respondents but likewise adjudges the property condemned in favor of petitioner over which parties, as well as their privies, are bound. Petitioner has occupied, utilized and, for all intents and purposes, exercised dominion over the property pursuant to the judgment. The exercise of such rights vested to it as the condemnee indeed has amounted to at least a partial compliance or satisfaction of the 1979 judgment, thereby preempting any claim of bar by prescription on grounds of nonexecution. In arguing for the return of their property on the basis of non-payment, respondents ignore the fact that the right of the expropriatory authority is far from that of an unpaid seller in ordinary sales, to which the remedy of rescission might perhaps apply. An in rem proceeding, condemnation acts upon the property. After condemnation, the paramount title is in the public under a new and independent title; thus, by giving notice to all claimants to a disputed title, condemnation proceedings provide a judicial process for securing better title against all the world than may be obtained by voluntary conveyance. Respondents, in arguing laches against petitioner did not take into account that the same argument could likewise apply against them. Respondents first instituted proceedings for payment against petitioner on 09 May 1984, or five years after the 1979 judgment had become final. The unusually long delay in bringing the action to compel payment against herein petitioner would militate against them. Consistently with the rule that one should take good care of his own concern, respondents should have commenced the proper action upon the finality of the judgment which, indeed, resulted in a permanent deprivation of their ownership and possession of the property. The constitutional limitation of just compensation is considered to be the sum equivalent to the market value of the property, broadly described to be the price fixed by the seller in open market in the usual and ordinary course of legal action and competition or the fair value of the property as between one who receives, and one who desires to sell, it fixed at the time of the actual taking by the government. Thus, if property is taken for public use before compensation is deposited with the court having jurisdiction over the case, the final compensation must include interests on its just value to be computed from the time the property is taken to the time when compensation is actually paid or deposited with the court. In fine, between the taking of the property and the actual payment, legal interests accrue in order to place the owner in a position as good as (but not better than) the position he was in before the taking occurred. The Bulacan trial court, in its 1979 decision, was correct in imposing interests on the zonal value of the property to be computed from the time petitioner instituted condemnation proceedings and took the property in September 1969. This allowance of interest on the amount found to be the value of the property as of the time of the taking computed, being an effective forbearance, at 12% per annum[28] should help eliminate the issue of the constant fluctuation and inflation of the value of the currency over time.[29] Article 1250 of the Civil Code, providing that, in case of extraordinary inflation or deflation, the value of the currency at the time of the establishment of the obligation shall be the basis for the payment when no agreement to the contrary is stipulated, has strict application only to contractual obligations.[30] In other words, a contractual agreement is needed for the effects of extraordinary inflation to be taken into account to alter the value of the currency.[31] All given, the trial court of Bulacan in issuing its order, dated 01 March 2000, vacating its decision of 26 February 1979 has acted beyond its lawful cognizance, the only authority left to it being to order its

execution. Verily, private respondents, although not entitled to the return of the expropriated property, deserve to be paid promptly on the yet unpaid award of just compensation already fixed by final judgment of the Bulacan RTC on 26 February 1979 at P6.00 per square meter, with legal interest thereon at 12% per annum computed from the date of "taking" of the property, i.e., 19 September 1969, until the due amount shall have been fully paid.

Anda mungkin juga menyukai